Đăng ký Đăng nhập
Trang chủ Giáo dục - Đào tạo Toán học Một số vấn đề về bất đẳng thức...

Tài liệu Một số vấn đề về bất đẳng thức

.PDF
75
396
89

Mô tả:

TRƯỜNG TRUNG HỌC PHỔ THÔNG CHUYÊN LÊ KHIẾT =====@@@===== MỘT SỐ VẤN ĐỀ VỀ BẤT ĐẲNG THỨC. TÁC GIẢ: NGUYỄN ANH KHOA LỚP : 10 TOÁN QUẢNG NGÃI, THÁNG 1 NĂM 2009 PDF created with pdfFactory Pro trial version www.pdffactory.com NGUYỄN ANH KHOA A.Lời giới thiệu: Các bất đẳng thức thuần nhất có điều kiện và không có điều kiện là hai bài toán hoàn toàn khác nhau như ẩn sau trong đó chúng lại có mối quan hệ mật thiết với nhau. Chính sự liên quan mật thiết này đã làm nảy sinh một kĩ thuật mới chứng minh bất đẳng thức đó là “kĩ thuật chuẩn hoá”. Trong bài viết này chúng ta sẽ khám phá kĩ thuật này có ý nghĩa như thế nào nhé! B. Kiến thức cơ bản: 1.1 Bất đẳng thức thuần nhất: Hàm số f ( x1 ; x2 ;...xn ) của các biến x1 ; x2 ;...xn được gọi là hàm thuần nhất bậc α nếu tồn tại số thực t thoả mãn: f (tx1 ; tx2 ;...txn ) = t α . f ( x1 ; x2 ;...xn ) Từ đó ta có định nghĩa bất đẳng thức thuần nhất như sau: * Bất đẳng thức thuần nhất là bất đẳng thức có dạng: f ( x1 ; x2 ;... xn ) ≥ 0 trong đó f ( x1; x2 ;... xn ) là hàm thuần nhất bậc α . Ví dụ: Bất đẳng thức AM-GM là bất đẳng thức thuần nhất bậc 1. tx1 + tx2 + ... + txn ≥ n n tx1.tx2 ....txn ⇒ x1 + x2 + ... + xn ≥ n n x1. x2 .... xn Từ đây trở đi trong bài viết này khi nói đến bất đẳng thức thuần nhất ta không cần quan tâm đến bậc α . 1.2 Một số phương pháp, kĩ thuật chứng minh bất đẳng thức: Trong phần này ta không xét hết tất cả các phương pháp, kĩ thuật chứng minh bất đẳng thức từ trước tới nay mà chỉ xét một số phương pháp sẽ được áp dụng trong bài viết này. 1.2.1 Phương pháp dồn biến: Phương pháp dồn biến tư tưởng chính là làm giảm số biến đã có thông qua các đại lượng trung bình, đưa bất đẳng thức cần chứng minh về dạng đơn giản hơn có thể chứng minh trực tiếp bằng cách khảo sát hàm một biến. Định lí dồn biến: Giả sử f ( x1 ; x2 ;...xn ) là một hàm số liên tục và đối xứng với tất cả các biến xác định trên một miền liên thong thoả mãn điều kiện sau: x +x x +x  f ( x1 , x2 ,....xn ) ≥ f  1 2 ; 1 2 ; x3 ;....xn  (1) 2  2  x + x + ... + xn Khi đó bất đẳng thức sau sẽ thoả mãn f ( x1 ; x2 ;... xn ) ≥ f ( x; x;....x ) trong đó x = 1 2 . n Điều kiện (1) có thể biến đổi thành một số dạng khác như: f ( x1 , x2 ,... xn ) ≥ f ( x1 x2 , x1 x2 , x3 ,...xn )  x2+x 2 x2+x 2  2 2 f ( x1 , x2 ,... xn ) ≥ f  1 , 1 , x3 ,... xn    2 2   Tuy nhiên trong bài viết này ta chỉ chú ý đến phương pháp dồn biến với các bất đẳng thức 3 biến nên ta sẽ xét đến trường hợp cụ thể như sau: Giả sử ta cần chứng minh: f ( x1 , x2 , x3 ) ≥ 0 ta có thể chứng minh: f ( x1 , x2 , xn ) ≥ f (t , t , x3 ) Trong đó giá trị của t có thể là : PDF created with pdfFactory Pro trial version www.pdffactory.com x1 + x2 2 + Trung bình nhân: t = x1 x2 + Trung bình cộng: t = x12 + x2 2 + Trung bình bình phương: t = 2 Sau đó ta chỉ cần chứng minh f (t , t , x3 ) ≥ 0 là bài toán được giải quyết. Chú ý khi chứng minh bất đẳng thức có điều kiện, ta thực hiện phép dồn biến thì phép dồn biến đó phải đảm bảo thoả mãn điều kiện của các biến; ví dụ như khi cho điều kiện là tổng thì ta chỉ dồn biến bằng trung bình cộng được mà thôi. Phương pháp dồn biến mà dựa vào trung bình cộng; trung bình nhân; trung bình bình phương là những dạng đơn giản nhất ngoài ra ta còn có một số phép dồn biến như sau: + Dồn biến toàn miền : sử dụng khi bất đẳng thức cần chứng minh có đại lượng chênh lệch bậc của các đại lượng xấp xỉ 0 ( x − y ), ( y − z ), ( z − x ) : f ( x, y , z ) ≥ f ( x − z , y − z , 0) + Dồn biến về biên: sử dụng khi đẳng thức xảy ra tại các giá trị biên. f ( x, y , z ) ≥ f (0, s , t ) trong đó s,t tuỳ thuộc vào mỗi bài toán + Dồn biến không xác định: (UMV) Nếu f là một hàm liên tục đối xứng xác định trên tập U  thoả điều kiện: f (...., xi ,...., x j ,...) ≥ min   x + xj x + xj    ,...., i ,....  ; f (...., 0,..., xi + x j ,....)  f  ...., i 2 2    Khi đó với mọi bộ x1 , x2 ,....xn ∈U thì f ( x1 , x2 ,...xn ) ≥ min {Ct }t = 0 , Nghĩa là GTNN của f ( x1 , x2 ,....xn ) n −1 sẽ đạt được khi và chỉ khi trong các số x1 , x2 ,....xn có t số bằng 0, các số còn lại bằng nhau. 1.2.2 Bất đẳng thức Schur và kĩ thuật đổi biến P,Q,R: a. Bất đẳng thức Schur: Với mọi số thực không âm a, b, c, k ta có: a k (a − b)(a − c) + b k (b − c )(b − a ) + c k (c − a )(c − b) ≥ 0 . Nếu k = 1 , thì ta có: a(a − b)(a − c) + b(b − c )(b − a ) + c(c − a )(c − b) ≥ 0 (1) (1) còn được xuất hiện ở các dạng sau: 1.(b + a − c)(b + c − a)(a + c − b) ≤ abc 2.(a + b + c )3 + 9abc ≥ 4(a + b + c )(ab + bc + ca ) 3.a 3 + b3 + c3 + 3abc ≥ ab(a + b) + bc(b + c) + ca (c + a) Nếu k = 1 , thì ta có: a 2 (a − b)(a − c) + b 2 (b − c )(b − a ) + c 2 (c − a)(c − b) ≥ 0 (2) (2) còn được xuất hiện ở dạng sau: a 4 + b 4 + c 4 + abc (a + b + c) ≥ a 3 (b + c) + b3 (c + a) + c 3 (a + b) * Chú ý: Bất đẳng thức Schur bậc 2 đúng với mọi số thực a, b, c . b. Bất đẳng thức Schur suy rộng:(Vornicu-Schur). Với các số thực dương a, b, c, x, y , z thoả mãn (a, b, c); ( x, y, z ) là các bộ đơn điệu. Khi đó ta có: x(a − b)(a − c) + y (b − c)(b − a) + z (c − a)(c − b) ≥ 0 Việc chứng minh bất đẳng thức Vornicu-Schur không khác gì cách chứng minh bất đẳng thức Schur nhưng các áp dụng của nó lại đa dạng và phong phú hơn bất đẳng thức Schur. Sau đây là một bất đẳng thức mạnh hơn bất đẳng thức Schur và công cụ chứng minh của nó là phải dùng tới phương pháp phân tích bình phương S.O.S: (bài toán sẽ được xét ở phần sau). a3 + b3 + c3 + 3abc ≥ ab 2(a 2 + b2 ) + bc 2(b 2 + c 2 ) + ca 2(c 2 + a 2 ) PDF created with pdfFactory Pro trial version www.pdffactory.com c. Kĩ thuật đổi biến P,Q,R: Định lí: Mọi đa thức đối xứng F (a, b, c ) đều có thể biểu diễn dưới dạng các đa thức đối xứng Viete. Nghĩa là có thể biểu diễn qua các đại lượng a + b + c, ab + bc + ca, abc . Từ đó ta có ý tưởng sau: Khi chứng minh một bất đẳng thức đối xứng ta có thể đổi biến lại như sau: Đặt p = a + b + c; q = ab + bc + ca; r = abc . Khi đó bất đẳng thức Schur bậc 0,1,2 được biểu diễn lại như sau: Với k = 0 thì pq − 9r ≥ 0 . (i) Với k = 1 thì (1) ⇔ p 3 − 4 pq + 9r ≥ 0 (ii) Với k = 2 thì (2) ⇔ p 4 − 5 p 2 q + 4q 2 + 6 pr ≥ 0 (iii). Trong thực hành ta thường sử dụng một số kết quả phân tích như sau: 1.ab(a + b) + bc (b + c) + ca (c + a) = pq − 3r 2.(a + b)(b + c)(c + a) = pq − r 3.(a + b)(a + c) + (b + c)(b + a) + (c + a )(c + b) = p 2 + q 4.ab(a 2 + b 2 ) + bc (b 2 + c 2 ) + ca (c 2 + a 2 ) = p 2 q − 2q 2 − pr 5.a 2 + b 2 + c 2 = p 2 − 2q 6.a 3 + b3 + c3 = p 3 − 3 pq + 3r 7.a 4 + b4 + c 4 = p 4 − 4 p 2 q + 2q 2 + 4 pr 8.a 2b 2 + b 2c 2 + c 2 a 2 = q 2 − 2 pr 9.a 3b3 + b3c3 + c 3a 3 = q 3 − 3 pqr + 3r 2 10.a 4b 4 + b 4c 4 + c 4 a 4 = q 4 − 4 pq 2 r + 2 p 2 r 2 + 4qr 2 Điều quan trọng mà ta rút ra được đó là (i);(ii);(iii) ta suy ra: pq r≤ 9  p(4q − p 2 )  r ≥ max 0,  4    (4q − p 2 )( p 2 − q)  r ≥ max 0,  6p   Đồng thời trong việc chứng minh ta cũng thường sử dụng một số bất đẳng thức ràng buộc giữa 3 biến p,q,r : p 2 ≥ 3q p 3 ≥ 27r q 2 ≥ 3 pr 2 p3 + 9r ≥ 7 pq p 2 q + 3 pr ≥ 4q 2 1.2.2 Look at the end point: Đây chính là kĩ thuật xét phần tử biên, trong bài viết này ta sẽ sử dụng một số định lí sau: Định lí 1: Nếu f ( x ) là hàm bậc nhất theo x thì nếu f (a) ≥ 0; f (b) ≥ 0 khi đó f ( x) ≥ 0 với mọi x ∈ [ a, b ] . PDF created with pdfFactory Pro trial version www.pdffactory.com Định lí 2: Nếu f ( x ) là hàm bậc nhất theo x thì : min { f (a); f (b)} ≤ f ( x ) ≤ max { f (a ); f (b)} với mọi x ∈ [ a, b ] . Định lí 3: Nếu f ( x ) là một hàm số lồi dưới trên khoảng [ a, b ] thì f ( x) ≤ max { f (a), f (b)} . Định lí 4: Nếu f ( x ) là một hàm số lõm dưới trên khoảng [ a, b ] thì f ( x ) ≥ min { f (a ), f (b)} . 1.2.3 Phương pháp cân bằng hệ số: 1.2.4 Kĩ thuật chọn điểm rơi đối với AM-GM: 1.2.5 Phương pháp phản chứng: Đây là một trong những ý tưởng khá hay trong việc chứng minh bất đẳng thức cũng như sáng tạo bất đẳng thức. Phương pháp này lấy ý tưởng từ bài toán sau: Bài toán: Cho hai hàm F ( x1 , x2 ,...xn ); G ( x1 , x2 ,...xn ) thuần nhất bậc α > 0 . Ta xét mệnh đề sau: Nếu F ( x1 , x2 ,...xn ) = k thì G ( x1 , x2 ,...xn ) ≥ m (*) ( k, m > 0) + Nếu F; G là hai hàm tăng đối với x1 , x2 ,...xn . Khi đó: (*) ⇔ Nếu G ( x1 , x2 ,...xn ) = m thì F ( x1 , x2 ,...xn ) ≤ k + Nếu F là hàm tăng; G là hàm giảm đối với x1 , x2 ,...xn . Khi đó: (*) ⇔ Nếu G ( x1 , x2 ,...xn ) = m thì F ( x1 , x2 ,...xn ) ≥ k Bạn đọc tự chứng minh bài toán này và nên ghi nhớ kết quả để sau này tiện sử dụng. C. Kĩ thuật chuẩn hoá bất đẳng thức thuần nhất đối xứng: Người ta sử dụng ý tưởng chuẩn hoá là như sau: Giả sử ta cần chứng minh bất đẳng thức thuần nhất f ( x1 , x2 ,...xn ) ≥ g ( x1 , x2 ,...xn ) trong đó f và g là hai hàm thuần nhất cùng bậc. do tính chất của hàm thuần nhất ta có thể chuyển bất đẳng thức trên về việc chứng minh bất đẳng thức f ( x1 , x2 ,... xn ) ≥ a với mọi x1 , x2 ,...xn thoả mãn g ( x1 , x2 ,... xn ) = a . Lợi ích của việc chuẩn hoá là ta có thể làm đơn giản các biểu thức của bất đẳng thức cần chứng minh, tận dụng được một số tính chất đặc biệt của hằng số. Bạn đọc có thể hiểu kĩ thuật chuẩn hoá thông qua bài toán sau. ab + bc + ca 3 (a + b)(b + c )(c + a ) ≤ (1) Problem: (STBĐT) CMR với a,b,c không âm thì 3 8 Chắc hẳn các bạn điều nhận ra rằng đây là bài toán từ sách “ Sáng tạo bất đẳng thức” của anh Phạm Kim Hùng cũng trong phần anh Hùng giới thiệu kĩ thuật chuẩn hoá. Vì thế tôi sẽ không đưa ra lời giải mà chỉ quan tâm tới cách thức chuẩn hoá, vì sao lại chuẩn hoá được. Hiển nhiên các bạn điều dễ dàng nhận ra bất đẳng thức cần chứng minh là thuần nhất. a b c Theo sách, anh Hùng chuẩn hoá ab + bc + ca = 3 . Khi ta lấy a ' = ; b ' = ; c ' = ta cần chọn t sao cho t t t ab + bc + ca a 'b ' + b 'c ' + c ' a ' = 3 lúc đó ta tìm được t = . Bât đẳng thức đúng với a ' , b ' , c ' nên nó sẽ 3 ' ' ' đúng với a,b,c sau khi nhân a , b , c với t. Như vậy việc tìm số t là xong ( tất nhiên các bước trên ta chỉ làm trong nháp không cần ghi vào bài làm).Bây giờ ta coi như chưa biết số t, ta sẽ tạo điều kiện a,b,c như sau PDF created with pdfFactory Pro trial version www.pdffactory.com 3 (a + b)(b + c )(c + a ) (ab + bc + ca)3 ≥1 : 8 27 (a + b)2 (b + c) 2 (c + a )2 ⇔ 3  ab + bc + ca  64.  3   3 ≥1 2 2           a b b c c a 1      ⇔3 + + + 8  ab + bc + ca ab + bc + ca   ab + bc + ca ab + bc + ca   ab + bc + ca ab + bc + ca      3 3 3 3 3 3      a b c Đặt x = ;y= ;z = khi đó ta có điều kiện xy + yz + xz = 3 và ab + bc + ca ab + bc + ca ab + bc + ca 3 3 3 bất đẳng thức cần chứng minh trở thành : ( x + y )( y + z )( x + z ) 3 ≥1 8 Có lẽ tới đây các bạn đã hiểu được vì sao ta lại chuẩn hoá được như vậy. Nhưng để tăng thêm niềm tin ta thử chuẩn hoá bài toán trên theo một cách khác thử xem. Chẳng hạn chuẩn hoá a + b + c = 1 a b c Ta đặt a ' = ; b ' = ; c ' = ta cần chọn t sao cho a ' + b ' + c ' = 1 , lúc đó ta tìmđược t = a + b + c . t t t Bây giờ ta xem như chưa biết số t, ta sẽ tạo điều kiện a,b,c như sau: (1) ab + bc + ca 1 (a + b)(b + c)(c + a) 1 . . ⇔ ≤3 2 3 3 8 (a + b + c) (a + b + c) 1  a 1  a + b   b + c  c + a  b b c c a  . . + . + . ≤ 3 .     3 a +b +c a +b +c a +b +c a +b+c a +b+c a +b+c 8  a + b + c   a + b + c  a + b + c  a b c Đặt x = ;y= ;z = khi đó ta có điều kiện x + y + z = 1 và bất đẳng thức cần a +b+c a+b+c a+b+c xy + yz + xz 3 ( x + y )( y + z )( x + z ) chứng minh trở thành: ≤ 3 8 Để hiểu sau hơn bạn đọc có thể tự mình chuẩn hoá bài toán trên theo abc = 1 hoặc (a + b)(b + c)(c + a) = 8 . Ghi chú: bất đẳng thức trên còn có một cách chứng minh khá hay như sau: Sử dụng hai bất đẳng thức phụ sau: (a + b)(b + c )(c + a ) (a + b + c )(ab + bc + ca) ≥ 8 9 2 (a + b + c) ≥ 3(ab + bc + ca ) Khi đó ta có: ⇔ 3 (a + b)(b + c )(c + a ) 3 (a + b + c )(ab + bc + ca) ≥ ≥ 8 9 3 3(ab + bc + ca )(ab + bc + ca ab + bc + ca = 9 3 PDF created with pdfFactory Pro trial version www.pdffactory.com 2    ≥    Cách giải này được GV Hoàng Đức Nguyên-khối THT chuyên ĐHSP Hà Nội đưa ra trong chuyên mục “bạn đọc tìm tòi” trong báo Toán hoc và tuổi trẻ với tên “Ứng dụng của một đẳng thức” Như vậy ta có thể thấy được một bất đẳng thức một khi đã thuần nhất thì có thể được chuẩn hoá bằng nhiều cách khác nhau. Chuẩn hoá là một kĩ thuật cơ bản nhưng kĩ thuật này lại đòi hỏi những kinh nghiệm và độ tinh tế nhất định. Đây cũng chính là điều độc đáo và khó khăn nhât của kĩ thuật này, vì chuẩn hoá một cách hợp lí thì ta mới có lời giải bài toán đơn giản nhất. Bây giờ chúng ta sẽ xem thử kĩ thuật chuẩn hoá có sức mạnh như thế nào trong thế giới bất đẳng thức. Bắt đầu từ đây trở đi trong mỗi bài toán ta sẽ không giải thích rõ ràng cách chuẩn hoá nữa mà điểm này sẽ dành cho bạn đọc. D. Kĩ thuật chuẩn hoá và ứng dụng: Trong phần bài tập tôi sẽ cố gắng ghi rõ nguồn gốc xuất xứ của bài toán từ đâu ra. Tuy nhiên do có một số sự hạn chế nên có một số bài toán chúng tôi không ghi rõ nguồn gốc xuất xứ mong bạn đọc thông cảm. Problem 1: ( England-1999) Cho x, y, z không âm. Chứng minh rằng: 2 ( x + y + z ) + 9 xyz ≥ 7 ( x + y + z )( xy + yz + xz ) 3 Solution: Bất đẳng thức đã cho là thuần nhất nên ta chuẩn hoá x + y + z = 1 , khi đó bất đẳng thức cần chứng minh trở thành: 2 + 9abc ≥ 7(ab + bc + ca) .(1) Do tính đối xứng của bất đẳng thức cần chứng minh nên ta hoàn toàn có thể giả sử x = max { x, y, z} . 1  Ta xét f ( x) = (7 y + 7 z − 9 yz ) x + 7 yz − 2 với x ∈  ,1 . 3  Sử dụng Look at the end point (định lí 1) ta có: 2 2 2   1  7( y + z ) + 12 yz − 6 4(9 yz − 1)  y+z  3 1 f  = = ≤ 0 vì yz ≤  = (2)  = 3 3  3  2   2  9   f (1) = 7( y + z ) − 2 yz − 2 = −2 < 0 vì x = 1 ⇒ y = z = 0 .(3) Từ (2)&(3) suy ra f ( x ) ≤ 0 ⇒ bất đẳng thức đã cho đúng. Đẳng thức xảy ra ⇔ x = y = z . Ta xét tiếp một bài toán tương tự sau: Problem 2: ( Sưu tầm). Cho x, y, z không âm. Chứng minh rằng: 9 xyz + ( x + y + z )3 ≥ 4( xy + yz + xz )( x + y + z ) Solution: Bất đẳng thức đã cho thuần nhất nên ta chuẩn hoá x + y + z = 1 . Khi đó bất đẳng thức cần chứng minh trở thành: 1 + 9 xyz ≥ 4( xy + yz + xz ) . Do bất đẳng thức có tính đối xứng nên ta giả sử x = max { x, y, z} . PDF created with pdfFactory Pro trial version www.pdffactory.com 1  Xét f ( x ) = (4 y + 4 z − 9 yz ) x + 4 yz − 1 với x ∈  ,1 . 3  Sử dụng Look at the end point (định lí1) ta có: 2 2 2  1  y+z  3 1  1  4( y + z ) + 3 yz − 3 f  = = yz − ≤ 0 vì yz ≤  = = .(2)  3 9  3  2   2  9   f (1) = 4( y + z ) − 5 yz − 1 = −1 < 0 vì x = 1 ⇒ y = z = 0 .(3) Từ (2)&(3) suy ra: f ( x ) ≤ 0 ⇒ bất đẳng thức đã cho đúng. Đẳng thức xảy ra ⇔ x = y = z . Comment 1: Qua hai bài toán trên chắc hẳn bạn đọc cũng đã thấy được sự hữu ích của việc chuẩn hoá. Việc chuẩn hoá không những làm cho bài toán nhìn đon giản hơn mà nó còn định hướng lời giải cho chúng ta một cách khá rõ ràng. Quả thật các bài toán từ nay trở về sau trong bài viết này nếu ta không làm một công việc là chuẩn hoá thì rất khó để cho một lời giải hay, đẹp trong từng bài toán được. Chú ý: Các bài toán trên điều có thể chứng minh một cách trực tiếp, bằng cách khai triển hai vế rút gọn sau đó sử dụng thêm BĐT Schur. Bằng phương pháp tương tự bạn đọc tự giải hai bài toán sau: Problem 3: (IMO-1984) Cho x, y, z không âm. Chứng minh rằng: 27( x + y + z )( xy + yz + xz ) ≤ 7( x + y + z )3 + 54 xyz Problem 4: ( Sưu tầm) Cho x, y, z không âm. Chứng minh rằng: 2( x + y + z )3 ≤ 9( x 3 + y 3 + z 3 ) + 27 xyz ≤ 9( x + y + z )3 . Gợi ý: Chuẩn hoá x + y + z = 1 . BĐT ở VT xảy ra tại biên. BĐT ở VP xảy ra tại tâm. Sau đây ta xét tiếp một lớp bài toán có mức độ khó khăn. Problem 5: (Macedonia 1999) Cho a, b, c không âm. Chứng minh rằng: abc(a + b + c) + (a 2 + b 2 + c 2 ) 2 ≥ 4abc 3(a 2 + b 2 + c 2 ) Solution: Bất đẳng thức đã cho thuần nhất và đồng thời với mong muốn biểu thức trong căn mất đi ta chuẩn hoá a 2 + b 2 + c 2 = 1 . Khi đó bất đẳng thức cần chứng minh trở thành: 1 abc(a + b + c ) + 1 ≥ 4 3.abc ⇔ a + b + c + ≥ 4 3 (1) abc Tới đây nếu giải theo cách thông thường ta sẽ thu được hai bất đẳng thức trái dấu đó là: PDF created with pdfFactory Pro trial version www.pdffactory.com a+b+c ≤ 3 1 ≥3 3 abc Vì thế ta phải tìm cách khác để giải quyết (1). Phương pháp tối ưu lúc này là sử dụng kĩ thuật chọn điểm rơi với bất đẳng thức AM-GM. Ta sẽ tìm cách tách a + b + c để khi sử dụng bất đẳng thức AM-GM dấu bằng sẽ xảy ra, đưa tham số α vào ta có: 1 αa + αb + αc + + (1 − α )(a + b + c) abc 1 nhưng do tính đối xứng của bài toán nên ta dự Sử dụng BĐT AM-GM dấu bằng xảy ra khi α a = abc 1 đoán dấu bằng xảy ra khi các biến bằng nhau, nghĩa là lúc đó ta có: α = 4 = 9 . Như vậy ta sẽ giải quyết a (1) như sau: 1 1 (1) ⇔ 9a + 9b + 9c + − 8(a + b + c) ≥ 4 4 93.abc. − 8 3(a 2 + b 2 + c 2 ) = 12 3 − 8 3 = 4 3 abc abc Vậy bất đẳng thức đã cho đúng. Đẳng thức xảy ra ⇔ a = b = c . Comment 2: Bất đẳng thức AM-GM là một bất đẳng thức thuần nhất nên nó rất hữu hiệu trong việc chứng minh các bất đẳng thức thuần nhất khác. Tuy nhiên điều khó khăn nhất của nó là điều kiện xảy ra dấu bằng rất nghiêm ngặt, vì thế việc áp dụng trực tiếp một cách máy móc rất dễ dẫn đến sai lầm. Bất đẳng thức AM-GM có khá nhiều kĩ thuật sử dụng nhưng bạn đọc nên biết 3 kĩ thuật chính: + Kĩ thuật cân bằng hệ số: sử dụng để giải các bất đẳng thức không đối xứng.(sẽ được giới thiệu ở phần sau) + Kĩ thuật chọn điểm rơi-trọng số: sử dụng để giải các bất đẳng thức đối xứng khi ta nhận thấy được dấu bằng xảy ra của bài toán. + Kĩ thuật AM-GM ngược dấu: sử dụng để giải các bất đẳng thức hoán vị. Sử dụng kĩ thuật trên ta giải các bài toán sau: Problem 6: (Crux 2946) Cho a, b, c dương. Chứng minh rằng: (ab + bc + ca)(a 2 + b 2 + c 2 ) + abc (a + b + c) ≥ 4abc 3(a 2 + b 2 + c 2 ) Solution: Bất đẳng thức đã cho là thuần nhất và cũng với mong muốn làm mất biểu thức trong căn nên ta chuẩn hoá a 2 + b 2 + c 2 = 1 . Khi đó bất đẳng thức cần chứng minh trở thành: ab + bc + ca + abc (a + b + c) ≥ 4 3.abc 1 1 1 ⇔ a + b + c + + + ≥ 4 3(1) a b c Sử dụng kĩ thuật chọn điểm rơi ta có: 1 1 1 1 (1) ⇔ 3a + 3b + 3c + + + − 2(a + b + c) ≥ 6 6 33.abc. − 2 3(a 2 + b 2 + c 2 ) = 6 3 − 2 3 = 4 3 a b c abc Vậy bất đẳng thức đã được chứng minh. Đẳng thức xảy ra ⇔ a = b = c . PDF created with pdfFactory Pro trial version www.pdffactory.com Problem 7: (Sưu tầm) Cho a, b, c dương. Chứng minh rằng: (ab + bc + ca)(a 2 + b 2 + c 2 ) ≥ 2abc 3(a 2 + b 2 + c 2 ) + abc (a + b + c) Solution: Bất đẳng thức đã cho thuần nhất ta chuẩn hoá a 2 + b 2 + c 2 = 1 . Khi đó bất đẳng thức cần chứng minh trở thành: ab + bc + ca ≥ 2 3.abc + abc(a + b + c ) 1 1 1 ⇔ + + − (a + b + c ) ≥ 2 3(1) a b c Sử dụng kĩ thuật chọn điểm rơi ta có: 1 1 1 1 (1) ⇔ + + + 3a + 3b + 3c − 4(a + b + c) ≥ 6 3 33.abc. − 4 3(a 2 + b 2 + c 2 ) = 6 3 − 4 3 = 2 3 a b c abc Vậy bất đẳng thức đã cho đúng. Đẳng thức xảy ra ⇔ a = b = c . Bạn đọc tự luyện các bài sau. Problem 8: (Sưu tầm) Cho a, b, c dương. Chứng minh rằng: 2(ab + bc + ca )(a 2 + b 2 + c 2 ) + 3abc(a + b + c ) ≥ 15abc ( a 2 + b 2 + c 2 ) 1 2 Problem 9: (Sưu tầm) Cho a, b, c dương. Chứng minh rằng: 3(ab + bc + ca )(a 2 + b 2 + c 2 ) + 4abc(a + b + c ) ≥ 21abc ( a 2 + b 2 + c 2 ) 1 2 Problem 10: (Sưu tầm) Cho a, b, c dương; m,n dương và 2n ≥ m . Chứng minh rằng: mabc(a + b + c) + n(ab + bc + ca)(a 2 + b 2 + c 2 ) ≥ 3(m + n)abc ( a 2 + b 2 + c 2 ) 1 2 Hướng dẫn: NX: Đây là bài toán tổng quát của các bài toán trên. Sau khi chuẩn hoá a 2 + b 2 + c 2 = 3 ta sẽ có được bất đẳng thức sau: 1 1 1 m(a + b + c ) + n  + +  ≥ 3(m + n) (*) a b c Ngoài cách sử dụng kĩ thuật chọn điểm rơi, ta có thể sử dụng cách giải sau: 1 1 1 Đặt X = + + ; Y = a + b + c a b c PDF created with pdfFactory Pro trial version www.pdffactory.com m m (Y + Y + X ) +  n −  X 2 2  2 Dễ dàng chứng minh: X ≥ 3; XY ≥ 27 . Khi đó : m m 9m m   T ≥ .3 3 XY 2 +  n −  .3 ≥ + 3  n −  = 3(m + n) 2 2 2 2   Tiếp theo đây ta sẽ xét những bài toán vừa áp dụng kĩ thuật chuẩn hoá vừa áp dụng phương pháp cân bằng hệ số. Gọi VT (*) là T ta có: T = Problem 11: (Sưu tầm) Cho a, b, c không âm. Chứng minh rằng: 289(a3 + 64b3 + c 3 ) ≥ 64(a + b + c)3 Solution: Rõ ràng đối với bài toán trên nếu ta khai triển hai vế dùng phương pháp biến đổi tương đương thì dẫn tới lời giải quá dài hoặc không ra. Nếu để ý ta thấy bất đẳng thức đã cho là thuần nhất nên nó định hướng cho ta nghĩ tới kĩ thuật chuẩn hoá bất đẳng thức. Nếu a + b + c = 0 thì bất đẳng thức hiển nhiên đúng. Nếu a + b + c > 0 ta chuẩn hoá a + b + c = 1 . Khi đó bất đẳng thức cần chứng minh trở thành: 64 a3 + 64b3 + c3 ≥ 289 Do vai trò a,c như nhau nên ta đưa vào đây các tham số α ; β : a3 + 64b3 + c 3 = ( a 3 + α 3 + α 3 ) + ( 64b3 + β 3 + β 3 ) + ( c 3 + α 3 + α 3 ) − 4α 3 − 2β 3 Áp dụng BĐT AM-GM trong từng dấu ngoặc ta có: a3 + 64b3 + c 3 ≥ 3aα 2 + 12bβ 2 + 3aα 2 − 4α 3 − 2 β 3 a = c = α  β  Đẳng thức xảy ra ⇔ b = ⇒ 8α + β = 4 (1) 4   a + b + c = 1 Đồng thời với ý muốn làm xuất hiện a + b + c = 1 nên ta phải có: 3α 2 = 12 β 2 ⇒ α = 2 β (2) 8 4 Từ (1)&(2) suy ra α = ; β = 17 17 64 Do đó: a3 + 64b3 + c3 ≥ . 289 Vậy bất đẳng thức đã cho đã được chứng minh. Đẳng thức xảy ra ⇔ a = c = 8b Cách khác: Ta dùng đạo hàm dồn biến. Do tính đối xứng của 2 biến a,c nên nó định hướng cho ta đánh giá qua một bất đẳng thức trung gian quen thuộc để từ đó ta xét theo ẩn b: a +c 3 3 (a + c) ≥ 4 3 ( ta dễ dàng chứng minh BĐT này bằng phương pháp biến đổi tương đương) Ta có: a + 64b + c ≥ 64b 3 3 3 3 (a + c) + 4 3 = 255b3 + 3b 2 − 3b + 1 4 PDF created with pdfFactory Pro trial version www.pdffactory.com 255b 3 + 3b 2 − 3b + 1 1 ∀b ∈ [ 0;1]) ⇒ f ' (b) = ( 765b 2 + 6b − 3) . ( 4 4 1 f ' (b) = 0 ⇒ b = . 17 64 . Lập bảng biến thiên của hàm số f (b) trên đoạn [ 0;1] ta thấy f (b) ≥ 289 64 Vậy a3 + 64b3 + c3 ≥ . Đẳng thức xảy ra ⇔ a = c = 8b . 289 Xét f (b) = Problem 12: (Sưu tầm) Cho a, b, c dương.Tìm GTNN : 10a 2 + 10b 2 + c 2 A= ab + bc + ca Soluiton: Biểu thức A có tính thuần nhất nên ta chuẩn hoá ab + bc + ca = 1 (1). Bài toán qui về tìm GTNN của A = 10a 2 + 10b 2 + c 2 với a,b,c thoả điều kiện (1). 2a 2 + 2b 2 ≥ 4ab 1 Ta có: 8a 2 + c 2 ≥ 4ac 2 1 8b 2 + c 2 ≥ 4bc 2 Cộng lại ta suy ra 10a 2 + 10b 2 + c 2 ≥ 4(ab + bc + ca ) = 4 . Đẳng thức xảy ra ⇔ 4a = 4b = c . Problem 13: (bài toán tổng quát). Cho a, b, c, k dương. Tìm GTNN của: A= k (a 2 + b 2 ) + c 2 ( k cho trước). ab + bc + ca Solution: Bất đẳng thức đã cho thuần nhất nên ta chuẩn hoá ab + bc + ca = 1 . Bài toán qui về tìm GTNN của A = k (a 2 + b 2 ) + c 2 với a,b,c thoả điều kiện trên. Ta tách k = l + (k − l ) ( 0 < l ≤ k ) . Áp dụng bất đẳng thức AM-GM ta có: la 2 + lb 2 ≥ 2lab 1 (k − l )a 2 + c 2 ≥ 2(k − l )ac 2 1 (k − l )b 2 + c 2 ≥ 2(k − l )bc 2 PDF created with pdfFactory Pro trial version www.pdffactory.com Với ý muốn xuất hiện ab + bc + ca = 1 thì ta phải có: 2l = 2(k − l ) ⇒ l = Vậy GTNN của A = −1 + 1 + 8k 4 1 −1 + 1 + 8k c . Đẳng thức xảy ra ⇔ a = b = 4 2(k − l ) Problem 14: (Sưu tầm). Cho a, b, c dương. Tìm GTLN của: 4ab + 6ac + 8bc A= ( a + b + c) 2 Solution: Biểu thức A có tính thuần nhất nên ta chuẩn hoá a + b + c = 1 . Bài toán qui về việc tìm GTLN của: A = 4ab + 6ac + 8bc với a,b,c thoả điều kiện trên. Ta sử dụng kĩ thuật hệ số bất định để tách ra các hạng tử trong biểu thức A như sau: A = α a (b + c ) + β b(a + c ) + γ c (a + b) = (α + β )ab + (α + γ )bc + ( β + γ )ca Đồng nhất các hệ số ta có: α + β = 4 α = 1   α + γ = 6 ⇔  β = 3 β + γ = 8 γ = 5   Do đó: 4ab + 6ac + 8bc = a (b + c) + 3b(a + c) + 5c(a + b) = a (1 − a ) + 3b(1 − b) + 5c (1 − c ) 2 2 2 9  1 1 1    = −  a −  + 3  b −  + 5  c −   4  2 2 2     1 1 1 1 1 1 3 1 ; y = b− ;z = c− ⇒ x+ y+ z = a− + b− + c− ≥ a+b+c− = 2 2 2 2 2 2 2 2 9 1 Bài toán qui về việc tìm GTLN của A = − ( x 2 + 3 y 2 + 5 z 2 ) với x + y + z ≥ . 4 2 Sử dụng phương pháp cân bằng hệ số ta có: 2  15  30 x2 +   ≥ x  46  46 Đặt x = a − 2 15 9 399  5  30 3 y + 3.   ≥ y ⇒ x2 + 3 y2 + 5z2 ≥ ⇒ A = − ( x 2 + 3 y 2 + 5z 2 ) ≤ 92 2 92  46  46 2 2  3  30 5 z + 5.  ≥ z  46  46 399 9 9 . Đẳng thức xảy ra ⇔ a = b = c . Vậy GTLN của A = 92 4 10 2 Bạn đọc tự luyện một số bài toán sau: PDF created with pdfFactory Pro trial version www.pdffactory.com Problem 15: (Sưu tầm) Cho a, b, c dương. Tìm GTNN của: a 3 + b3 + 16c 3 A= ( a + b + c )3 Problem 16: (Sưu tầm) Cho a, b, c, d dương. Tìm GTNN của: A= 5a 2 + 4b 2 + 5c 2 + d 2 ab + bc + cd + da Problem 17: (Sưu tầm) Cho a, b, c dương. Tìm GTLN của: ab + 2bc + 3ac A= (a + b + c )2 Comment 3: Kĩ thuật cân bằng hệ số là một kĩ thuật cần thiết và thường được sử dụng, mặc dù đôi lúc ta phải giải quyết nhiều hệ phương trình khá phức tạp. Nhưng đối với các bài toán không ở dạng chuẩn tức là không đối xứng, không hoán vị, các biểu thức lệch nhau thì công việc này dường như là bắt buộc. Qua các bài toán trên ta càng thấy tầm ứng dụng quan trọng của kĩ thuật chuẩn hoá, quả thật các bài toán trên nếu ta không kèm theo sử dụng kĩ thuật chuẩn hoá thì không thể giái quyết chúng được. Tiếp theo ta sẽ xét một số bài toán vừa sử dụng kĩ thuật chuẩn hoá vừa sử dụng phương pháp dồn biến. Problem 18: (Vasile Cirtoaje) Cho x, y, z dương. Chứng minh rằng: 3 1 4 3 2 7 3 x + y 2 + z 2 ) 2 + 9 xyz ≥ ( xy + yz + zx ) ( x2 + y 2 + z 2 ) 2 ( 3 3 Solution: Ngoại hình của bài toán thật “cồng kềnh” nếu như theo lói mòn ta suy nghĩ sử dụng phương pháp biến đổi tương đương như bình phương hai vế chẳng hạn hay sử dụng phương pháp mạnh như S.O.S hay dồn biến thì vẫn không dễ gì ra bài toán. Nhưng ta hãy bình tĩnh suy xét lại cấu hình của nó. Dễ nhận thấy bất đẳng thức đã cho thuần nhất và với mong muốn làm mất đi các biểu thức trong căn ta chuẩn hoá x 2 + y 2 + z 2 = 3 .Khi đó bất đẳng thức cần chứng minh trở thành: 12 + 9 xyz ≥ 7( xy + yz + zx ) Đến đây ta sử dụng phương pháp dồn biến theo trung bình bình phương. Ta chứng minh: PDF created with pdfFactory Pro trial version www.pdffactory.com  x2 + y 2 x 2 + y 2  f ( x, y , z ) − f  , ,z ≤ 0   2 2   2 14 z ( x − y ) ⇔ (9 z − 7)( x − y )2 ≤ x + y + 2( x 2 + y 2 ) 7  Ta cần chứng minh: (9 z − 7) 2( x 2 + y 2 ) ≤ 7 z ⇔  9 −  2(3 − z 2 ) ≤ 7 . BĐT này đúng do z ≤ 1 . z  Cuối cùng ta cùng ta cần chứng minh trong trường hợp x = y . Đặt t = x2 + y 2 , BĐT cần chứng minh 2  3 − z2  3 − z2  3 − z2  trở thành: 7(t 2 + 2 zt ) ≤ 12 + 9t 2 z ⇔ 7  + 2z  ≤ 12 + 9 z    2 2   2   Công việc của chúng ta bây giờ là khảo sát hàm số biến z. (bạn đọc tự giải) Cách khác: Ta dồn tất cả về một biến z như sau.BĐT cần chứng minh tương đương với BĐT sau: 7 z ( x + y ) + xy (7 − 9 z ) − 12 ≤ 0 BĐT trên đúng thật vậy : (7 − 9 z )( x 2 + y 2 ) (7 − 9 z )(3 − z 2 ) 7 z (5 − z 2 ) 7 z ( x + y ) + xy (7 − 9 z ) ≤ + 7 z 2( x 2 + y 2 ) ≤ + − 12 2 2 2 3  ( z − 1)2  z −  2 2 (7 − 9 z )(3 − z ) 7 z (5 − z ) 2  + = ≤ 0 ( z ∈ [0,1]) . Đặt f ( z ) = 2 2 2 Vậy bất đẳng thức đã được chứng minh. Đẳng thức xảy ra ⇔ x = y = z . Commnet 4: Thật ra với bài toán này thì phương pháp dồn biến vẫn chưa phải là tối ưu nhất, bởi vì khảo sát hàm z ở cách 1 hơi khó. Nhưng vẫn với ý tưởng dồn tất cả về một biến ta đã giải quyết bài toán theo cách 2 một cách nhẹ nhàng, ngắn gọn. Ta đã vận dụng một cách khéo léo BĐT AM-GM khi đánh giá: 2 + x2 + y 2 5 − z2 2( x 2 + y 2 ) ≤ = . 2 3 Do ta dự đoán x = y = 1 nên ta áp dụng BĐT AM-GM giữa hai số 2 và x 2 + y 2 mà vẫn đảm bảo điều kiện dấu bằng xảy ra. Problem 19: (Việt Nam MO-2002). Cho x, y, z là số thực bất kì. Chứng minh rằng: 6( x + y + z )( x 2 + y 2 + z 2 ) ≤ 27 xyz + 10 ( x 2 + y 2 + z 2 ) 3 2 Solution: Bất đẳng thức thuần nhất nên ta chuẩn hoá: x 2 + y 2 + z 2 = 9 . Khi đó bất đẳng thức cần chứng minh trở thành: 2( x + y + z ) ≤ xyz + 10 ⇔ [ 2( x + y + z ) − xyz ] ≤ 100 . 2 Không mất tính tổng quát giả sử x ≤ y ≤ z .Áp dụng BĐT Cauchy-Schwarz: [ 2( x + y + z ) − xyz ] 2 2 =  ( x + y ) 2 + z (2 − xy )  ≤ ( x + y )2 + z 2   2 2 + (2 − xy )2  = 72 − 20 xy + x 2 y 2 + 2 x 3 y 3 PDF created with pdfFactory Pro trial version www.pdffactory.com ⇔ [ 2( x + y + z ) − xyz ] ≤ 100 + ( xy + 2) 2 (2 xy − 7) . 2 Từ x ≤ y ≤ z ⇒ z 2 ≥ 3 ⇒ 2 xy ≤ x 2 + y 2 ≤ 6 . Suy ra đpcm. Cách khác: Ta sử dụng phương pháp dồn biến . Xét hiệu:   y2 + z2 y 2 + z2  x( y − z ) 2 2 x f  x, , = ( y − z )2  −   − f ( x, y , z ) = 2 2( y 2 + z 2 ) − y − z −   2( y 2 + z 2 ) + y + z 2  2 2  2    Nếu x, y, z > 0 . Ta xét hai trường hợp: ) ( 1 ≤ x ≤ y ≤ z . Khi đó: 2( x + y + z ) − xyz ≤ 2 3( x 2 + y 2 + z 2 ) − 1 = 6 3 − 1 < 10 . 0 ≤ x ≤ 1 . Khi đó: 2( x + y + z ) − xyz < 2( x + y + z ) ≤ 2 x + 2 2( y 2 + z 2 ) = 2 x + 2 2(9 − x 2 ) = g ( x) ⇒ g ' ( x) = 2 − 2 x 9 − x2 > 0 ⇒ g ( x) ≤ g (1) = 10 .  y2 + z2 y 2 + z2  Nếu trong 3 số x, y, z có một số âm, giả sử x < 0 . Khi đó: f  x, ,  − f ( x, y, z ) ≥ 0 .   2 2   2 2 2 2 2  x(9 − x ) y +z y +z  Ta cần chứng minh f  x, , ≤ 10  ≤ 10 hay : 2 x + 2 2(9 − x 2 ) −   2 2 2   4x 2 ⇔ h( x) = x3 − 5 x + 4 2(9 − x 2 ) ≤ 20 . Ta có: h' ( x) = 3x 2 − 5 − = 0 ⇔ x = −1 ⇒ h( x) ≤ h(−1) = 20 9 − x2 Problem 20: (Nguyễn Anh Khoa) Cho x, y, z dương. Chứng minh rằng: 3 3(a 2 + b 2 + c 2 ) + ( a + b + c )2  ≥ 5(a + b + c)6 + 54abc(a + b + c )3 + 729a 2b 2 c 2   Solution: Bất đẳng thức đã cho thuần nhất nên ta chuẩn hoá a + b + c = 3 . Khi đó bất đẳng thức cần chứng minh 3 trở thành:. 3(a 2 + b 2 + c 2 ) + 9  ≥ 5.36 + 1458abc + 729a 2b 2 c 2 3 ⇔ 3 + a 2 + b 2 + c 2  ≥ 135 + 54abc + 27 a 2b 2c 2 Sử dụng bất đẳng thức AM-GM ta có: ( 3 + a 2 + b 2 + c 2 ) ≥ 27(1 + a 2 )(1 + b 2 )(1 + c 2 ) 3 Khi đó ta phải chứng minh : (1 + a 2 )(1 + b 2 )(1 + c 2 ) ≥ 5 + abc(2 + abc) ⇔ a 2 + b 2 + c 2 + a 2b 2 + b 2 c 2 + c 2 a 2 ≥ 4 + 2abc(1) Lại có: a 2b 2 + b 2 c 2 + c 2 a 2 ≥ abc(a + b + c) = 3abc .(2) Từ (1)&(2) ta phải chứng minh BĐT sau: a 2 + b 2 + c 2 + abc ≥ 4 Ta viết BĐT cần chứng minh dưới dạng: (b + c) 2 − 2bc + a 2 + abc − 4 ≥ 0 ⇔ (3 − a )2 + a 2 + bc(a − 2) − 4 ≥ 0 ⇔ (a − 2)bc + 2a 2 − 6a + 5 ≥ 0 PDF created with pdfFactory Pro trial version www.pdffactory.com   (3 − a) 2   Ta cố định a xét hàm bậc nhất f (bc ) = (a − 2)bc + 2a − 6a + 5  bc ∈ 0;  4    2  (3 − a )2  Theo tính chất về hàm bậc nhất thì f (bc ) ≥ min{ f (0); f  }  4  2 3 1  Mà: f (0) = 2a − 6a + 5 = 2  a −  + > 0 2 2   (3 − a) 2  (3 − a) 2 1 f a = ( − 2). + 2a 2 − 6a + 5 = (a − 1) 2 (a + 2) ≥ 0  4 4  4  Do đó f (bc ) ≥ 0 Vậy bất đẳng thức đã được chứng minh. Đẳng thức xảy ra ⇔ a = b = c = 1 . Sau đây là một số bài toán sử dụng thêm kĩ thuật đổi biến p,q,r. 2 Problem 21: (Phạm Sinh Tân) Cho a, b, c không âm và không có hai số nào đồng thời bằng 0. CMR với mọi k ≥ 1 ta luôn có: a b c (a + b + c)(ab + bc + ca) + + + k. ≥ 2 k +1 b+c c+a a +b a 3 + b3 + c 3 Solution: Bất đã cho là thuần nhất nên ta chuẩn hoá: a + b + c = 1 . Khi đó bất đẳng thức cần chứng minh trở thành: a b c ab + bc + ca + + + k. 3 3 3 ≥ 2 k + 1. b+c c+a a +b a +b +c Đổi biến a, b, c theo p, q, r ; khi đó ta cần chứng minh: 1 − 2q + 3r q + k. ≥ 2 k +1 . q−r 1 − 3q + 3r q q q 1 − 2q + 3r 1 − 3q + 3r 1 − 3q + 3r + k. = + k. +1 ≥ + k. +1 Ta có: q−r 1 − 3q + 3r q−r 1 − 3q + 3r q 1 − 3q + 3r 1 − 3q + 3r q + k. + 1 ≥ 2 k + 1. Sử dụng tiếp bất đẳng thức AM-GM ta có: q 1 − 3q + 3r  k + 2 k − 3 + k +1  Đẳng thức xảy ra ⇔ ( a, b, c ) =  x, x, 0  hoặc các hoán vị.   2   Problem 24: (Dương Đức Lâm). Cho a, b, c không âm và không có hai số nào đồng thời bằng 0. CMR: 2 2 2 10abc  a   b   c  ≥2   +  +  +  b + c   c + a   a + b  (a + b)(b + c )(c + a ) Solution: PDF created with pdfFactory Pro trial version www.pdffactory.com 2a 2b 2c ;y= ;z = khi đó ta có: xy + yz + xz + xyz = 4 . Bất đẳng thức cần chứng minh trở b+c c+a a +b thành: x 2 + y 2 + z 2 + 5 xyz ≥ 8 . Đổi biến x, y, z theo p, q, r khi đó giả thiết: q + r = 4 . BĐT cần chứng minh trở thành: p 2 − 2q + 5r ≥ 8 ⇔ p 2 − 7 q + 12 ≥ 0 . Đặt x = Nếu 4 ≥ p sử dụng bất đẳng thức Schur r ≥ p(4q − p 2 ) p (4q − p 2 ) p 3 + 36 ⇒4≥q+ ⇔q≤ 9 9 4p +9 7( p 3 + 36) Do đó: p − 7q + 12 ≥ p − + 12 . Ta cần chứng minh: 4p+9 7( p3 + 36) p2 − + 12 ≥ 0 ⇔ ( p − 3)( p 2 − 16) ≤ 0 . 4p+9 2 2 Bất đẳng thức trên đúng vì: 4 ≥ q ≥ 3q ≥ 3 . p2 ≥ 8. 2 Vậy bất đẳng thức đã được chứng minh. Đẳng thức xảy ra khi x = y = z = 1 hoặc x = y = 2, z = 0 và các hoán vị tương ứng. Comment 5: Bài toán trên khá hay ta đã đặt một lần ẩn phụ rồi sau đó ta mới đổi biến theo p,q,r. Đến đây ta đã dung một thủ thuật rất hay dùng khi sử dụng bất đẳng thức Schur đó là chia trường hợp ra để giải quyết. Bài toán trên trong khi sử dụng phương pháp phân tích bình phương S.O.S là khá dài dòng nhưng ta đã có một lời giải đẹp gọn gàng thoả mãn mỹ quan về mặt toán học khi sử dụng khéo léo kĩ thuật đổi biến p,q,r. Sau đây là một số bài toán tương tự như bài trên. Nếu p ≥ 4 ta có: p 2 ≥ 16 ≥ 4q nên: p 2 − 2q + 5r ≥ p 2 − 2q ≥ Problem 26: (Toán học&Tuổi trẻ). Cho a, b, c là các số không âm và không có hai số nào đồng thời bằng 0. CMR: a b c 4abc + + + ≥2 b + c c + a a + b (a + b)(b + c)(c + a ) Problem 27: (Toán tuổi thơ) Cho a, b, c là các số dương. Chứng minh rằng: a b c 2ab 2bc 2ca + + ≥ + + b + c c + a a + b (b + c )(c + a ) (c + a)(a + b) (b + c)(a + b) Ta xét tiếp một bài toán kinh điển sau: Problem 28: (Iran-1996) Cho x, y, z không âm và không có hai số nào đồng thời bằng 0. CMR:  1 1 1  9 ( xy + yz + xz )  + + ≥ 2 2 2  ( x + y) ( y + z ) ( x + z )  4 PDF created with pdfFactory Pro trial version www.pdffactory.com Solution: Do tính đối xứng của bài toán nên ta giả sử x ≥ y ≥ z . Sử dụng bất đẳng thức Cauchy-Schwarz ta có: 1 1 1 1 1  ( x + y + 2z)2 + ≥  + = ( y + z ) 2 ( x + z )2 2  x + y x + z  2( y + z )2 ( x + z ) 2 Nên ta cần chứng minh:  1 ( x + y + 2 z) 2  9 ( xy + yz + xz )  + ≥ 2 2 2  ( x + y ) 2( x + z ) ( y + z )  4 Do tính thuần nhất của bất đẳng thức ta chuẩn hoá x + y = 1 và đặt a = xy ⇒ 1 ≥ a ≥ z (1 − z ) . Khi đó bất 4 đẳng thức trở thành: (1 + 2 z ) 2 (a + z ) 9 − ≥0 f ( a) = a + z + 2( z + z 2 + a )2 4 (1 + 2 z )2 ( z + a − z 2 ) '' (1 + 2 z ) 2 ( z − 2 z 2 + a ) Ta có: f ' (a) = 1 − ; f ( a ) = ≥0 2( z + z 2 + a)3 ( z + z 2 + a )4 3 2  1  (2 z − 1)(8 z + 20 z + 38 z + 7) Nên f ' (a) đồng biến ta suy ra: f ' (a) ≤ f '   = ≤ 0. (2 z + 1)4 4 2  1  z (1 − 2 z ) Do đó f (a ) nghịch biến vậy nên: f (a) ≥ f   = ≥ 0. 2  4  (1 + 2 z ) Vậy bất đẳng thức đã được chứng minh. Đẳng thức xảy ra khi x = y = z hoặc x = y = 1, z = 0 . Comment 6: Bài toán trên là một trong những bài toán khá nổi tiếng trong làng bất đẳng thức và đã có khá nhiều cách xuất hiện để giải quyết nó như: phương pháp S.O.S, phương pháp dồn biến, phương pháp p,q,r…Nhưng cách giải trên khá mới mẻ mà người ta gọi đó là: “Kĩ thuật Cauchy bất đối”. Bạn đọc có thể tìm hiểu kĩ hơn kĩ thuật này trong bài viết cùng tên của anh Võ Quốc Bá Cẩn. Problem 29: (Phạm Kim Hùng). Cho a, b, c dương. Chứng minh rằng: a b c 3 3 abc + + + ≥2 b + c c + a a + b 2(a + b + c) Solution: Sử dụng bất đẳng thức Cauchy-Schwarz ta có: a b c 3 3 abc ( a + b + c) 2 3 3 abc + + + ≥ + b + c c + a a + b 2(a + b + c) 2(ab + bc + ca ) 2(a + b + c ) Do tính thuần nhất của bất đẳng thức nên ta chuẩn hoá: abc = 1 . Đổi biến a, b, c theo p, q, r ta có bất đẳng thức tương đương: p2 3 + ≥ 2 ⇔ p 3 + 3q ≥ 4 pq ⇔ ( p3 − 4 pq + 9r ) + (3q − 9r ) ≥ 0 2q 2 p Bất đẳng thức trên đúng do: p 3 − 4 pq + 9r ≥ 0 ( Bất đẳng thức Schur bậc 1) 3(ab + bc + ca ) ≥ 9 2 a 2b 2c 2 = 9 = 9abc ⇔ 3q − 9r ≥ 0 PDF created with pdfFactory Pro trial version www.pdffactory.com Vậy bất đẳng thức đã được chứng minh. Đẳng thức xảy ra ⇔ a = b = c . Problem 30: (Olympic Ba Lan-2005) Cho a, b, c dương. Chứng minh rằng: 3 3 a3 + b3 + c3 + 6abc ≥ ( ab + bc + ca ) 2 3 Solution: Bất đẳng thức đã cho là thuần nhất nên ta chuẩn hoá: ab + bc + ca = 3 . Khi đó bất đẳng thức cần chứng minh trở thành: a3 + b3 + c 3 + 6abc ≥ 9 Trước hết ta chứng minh bất đẳng thức sau: a3 + b3 + c 3 + 7 abc ≥ 10 . Đổi biến a, b, c theo p, q, r ta có:  p(4q − p 2 )   p (12 − p 2 )  Áp dụng bất đẳng thức Schur ta có: r ≥ max 0,  = max 0, . 9 9     3 Ta cần chứng minh: p − 9 p + 10r ≥ 10 . Đến đây ta dùng thủ thuật “Chia để trị” để giải quyết: Nếu p ≥ 2 3 thì: p 3 − 9 p + 10r ≥ p3 − 9 p − 10 ≥ 12 p − 9q − 10 = 3 p − 10 > 0 . Nếu p ≤ 2 3 < 4 thì: 10 1 p (12 − p 2 ) − 10 = ( p − 3) (16 − p 2 ) + 3(4 − p ) + 2 ≥ 0 9 9 3 3 3 3 3 3 Như vậy a + b + c + 7 abc ≥ 10 ⇔ a + b + c + 6abc ≥ 9 + 1 − abc ≥ 9 . p 3 − 9 p + 10r − 10 ≥ p 3 − 9 p + Bất đẳng thức trên đúng do: 3 = ab + bc + ca ≥ 3 3 a 2b 2c 2 ⇔ abc ≤ 1 . Đẳng thức xảy ra ⇔ a = b = c . Problem 31: (Sưu tầm) Cho a, b, c dương. Chứng minh rằng: 1 1 1 9 1 1   1 + + + ≥ 4 + +  a b c a+b+c a +b b+c c+a  Solution: Bất đẳng thức đã cho là thuần nhất nên ta chuẩn hoá a + b + c = 1 . Khi đó bất đẳng thức cần chứng minh tương đương với bất đẳng thức sau: 1 1 1 4 4 4 + + +9≥ + + .(1) a b c a+b b+c c+a x y z ;b = ;c = Đặt a = . Khi đó (1) tương đương: x+ y+z x+ y+ z x+ y+ z x+ y+ z x+ y+ z x+ y+ z 4( x + y + z ) 4( x + y + z ) 4( x + y + z ) + + +9 ≥ + + x y z x+ y y+z x+z  x y+ z x+ z x+ y y z  + + ≥ 4 + +  (2) x y z  y+ z x+ z x+ y 1 1 4 Ta sử dụng bất đẳng thức phụ: + ≥ . Ta có: A B A+ B ⇔ PDF created with pdfFactory Pro trial version www.pdffactory.com
- Xem thêm -

Tài liệu liên quan